Đến nội dung

Phạm Hữu Bảo Chung nội dung

Có 549 mục bởi Phạm Hữu Bảo Chung (Tìm giới hạn từ 26-04-2020)



Sắp theo                Sắp xếp  

#334347 Giải PT : $(8sin^{3}x+1)^{3}-162 sinx +27= 0$

Đã gửi bởi Phạm Hữu Bảo Chung on 11-07-2012 - 11:59 trong Phương trình, Hệ phương trình Lượng giác

Em chưa học Toán 11 nên chưa học phần arc. Nhưng em nghĩ là $\tan{x} = \pm 1 \Rightarrow \sin{x} = \pm \cos{x}$
Thế vào $\cos^2{x} + \sin^2{x} = 1$ thì ta được các nghiệm nói trên :)




#317450 Tìm m để hệ sau có nghiệm thực duy nhất

Đã gửi bởi Phạm Hữu Bảo Chung on 17-05-2012 - 21:43 trong Đại số

Ừ, chỉ có 1 nghiệm S thỏa mãn đề bài. Nên nó cũng chỉ có một nghiệm P thỏa mãn.
Ở đây người ta yêu cầu hệ có nghiệm duy nhất, tức là một cặp số (x; y) duy nhất chứ không phải một tổng x + y duy nhất.

Không liên quan nhiều đến bài này nhưng mà xem một chút nhé:
Chẳng hạn: Ta tìm được:
x + y = 6 là duy nhất (S = const ) nhưng...
có biết bao nhiêu cặp số thỏa mãn điều này? Có tới hàng ngàn, hàng triệu, hàng tỉ số thỏa mãn:
(0; 6); (6; 0); (5; 1); (4; 2); (3; 3)…….(1006; -1000)….
Các cặp số đó có tích khác nhau, do đó, ta cần phải cố định tích của chúng. Chẳng hạn tích của chúng là 5. Vậy bây giờ, có 2 cặp thỏa mãn điều này: (1; 5) và (5; 1)
Vì thế tích của chúng bằng 5 không thể giúp hệ ban đầu có cặp nghiệm duy nhất.
Với tích bằng 9, dễ thấy, chỉ có một nghiệm thỏa mãn đề bài là (x; y) = (3; 3). Vậy tích bằng 9 thỏa mãn yêu cầu.
Vậy thì làm sao bây giờ? Nếu không có một nghiệm S duy nhất thì chắc chắn là không thể có một cặp nghiệm duy nhất rồi !? Chẳng lẽ phải ăn may sao. Phải tìm được S duy nhất, tìm được một P thỏa mãn có nghiệm duy nhất. Đáng tiếc rằng, trên đời, chỉ có được vài ba bài toán hên như thế và có cả đống bài toán không thể làm điều này, cụ thể như bài toán ban đầu…

Nhớ rằng chúng ta bỏ quên mất một điều kiện là: Để tồn tại x, y thỏa mãn x + y = S; xy = P thì $S^2 \geq 4P$
Do đó, chẳng hạn ta tìm được 2 S phân biệt. Từ đó ta dễ dàng có 2 giá trị P.
Nhưng…chỉ có một cặp trong số đó thỏa mãn điều kiện: $S^2 \geq 4P$, cặp còn lại thì không! Đành ngậm ngùi dứt áo ra đi vậy J
Vì vậy, ta vẫn có quyền hi vọng rằng trong 2 cặp số nào đó. Có một cặp (S; P) cho nghiệm duy nhất (x; y) (Khi đó thì x = y) và cặp còn lại không thỏa mãn đề bài!

P/S: Chủ topic, bước thứ 2 kể từ chỗ: Điều kiện để hệ có nghiệm...Tớ đã bỏ đi (làm tắt) một trường hợp đấy :).
Ai bảo vô nghiệm, nên tớ bỏ:

$\sqrt{\dfrac{m + 6}{3}} \geq 0 > -2 - \sqrt{m + 4}$



#381050 Giải phương trình a) $(x-3)^{2}+x^{4}=-y^{2...

Đã gửi bởi Phạm Hữu Bảo Chung on 27-12-2012 - 22:05 trong Phương trình, hệ phương trình và bất phương trình

Giải

ĐK: $x \neq 0$

Ta có:
$x^4 + \dfrac{8}{x^2} + y^2 = 4 + 2x^2$


$\Leftrightarrow y^2 = (4 - x^4) + (2x^2 - \dfrac{8}{x^2})$

$\Leftrightarrow y^2 = (4 - x^4) + \dfrac{2}{x^2}(x^4 - 4) = (x^4 - 4)(\dfrac{2}{x^2} - 1)$

$\Leftrightarrow y^2 = -\dfrac{(x^2 + 2)(x^2 - 2)^2}{x^2}$

Nhận thấy: $VT \geq 0 \geq VF$

Dấu "=" xảy ra khi:

$\left\{\begin{matrix}y = 0\\x^2 = 2\end{matrix}\right.$



#431723 Giải pt lượng giác: $(\frac{tanx-1}{tanx+1}+co...

Đã gửi bởi Phạm Hữu Bảo Chung on 30-06-2013 - 00:08 trong Phương trình, Hệ phương trình Lượng giác

\[2\left[ {\sin 3x.\sin x + \sin \left( {3x + \frac{{5\pi }}{2}} \right).\sin \left( {x + \frac{\pi }{2}} \right)} \right] = 1\,\,\,\,\left( { - \pi  < x < \pi } \right)\]

 

 

Giải

Phương trình ban đầu tương đương: 

$2 \left [ \sin{3x} \sin{x} + \cos{3x}\cos{x} \right ] = 1$

$\Leftrightarrow 2.\cos{2x} = 1 \Leftrightarrow x = \pm \dfrac{\pi}{6} + k\pi \, (k \in Z)$

 

Do $- \pi < x < \pi$ nên $x = \dfrac{\pi}{6}; \dfrac{-\pi}{6}; \dfrac{5\pi}{6}; \dfrac{-5\pi}{6}$

 

\[\cos 2x + \sin \left( {2x + \frac{\pi }{3}} \right) + \sin \left( {2x - \frac{\pi }{3}} \right) = \sqrt 2 \sin \left( {x - \frac{\pi }{3}} \right)\]

 

Giải

Phương trình ban đầu tương đương:
$\cos{2x} + 2\sin{2x}\cos{\dfrac{\pi}{3}} = \sqrt{2}\sin{\left ( x - \dfrac{\pi}{3}\right )}$

 

$\Leftrightarrow \dfrac{1}{\sqrt{2}}(\sin{2x} + \cos{2x}) = \sin{\left ( x - \dfrac{\pi}{3}\right )}$

 

$\Leftrightarrow \sin{\left (2x + \dfrac{\pi}{4} \right )} = \sin{\left ( x - \dfrac{\pi}{3}\right )}$

 

Phương trình lượng giác cơ bản.




#452920 $\sqrt[3]{x^{2}-2}= \sqrt{2-x^{3...

Đã gửi bởi Phạm Hữu Bảo Chung on 25-09-2013 - 12:04 trong Phương trình - hệ phương trình - bất phương trình

Giải

ĐK: $- \sqrt[3]{2} \leq x \leq \sqrt[3]{2}$

Do $\left [- \sqrt[3]{2}; \sqrt[3]{2}\right ] \subset \left [- \sqrt{2}; \sqrt{2}\right ]$

Vì vậy: $x^2 - 2 < 0 \Rightarrow \sqrt[3]{x^2 - 2} < 0 < \sqrt{2 - x^3}$

Phương trình đã cho vô nghiệm.

 

 




#448394 Giải phương trình $\sqrt{7-x^2+x\sqrt{x+5}...

Đã gửi bởi Phạm Hữu Bảo Chung on 07-09-2013 - 12:30 trong Phương trình - hệ phương trình - bất phương trình

Hì! Đáng lẽ ra nên làm thế này. Mình làm ra cho dễ hiểu ^^

Ta có: $1 = 2.\dfrac{1}{2} = 2\cos{\dfrac{\pi}{3}} = \cos{\dfrac{\pi \pm 6\pi}{3}}$

Nhận xét rằng:
$2\cos{\dfrac{\pi}{3}} = 2\cos{3.\dfrac{\pi}{9}} = 8\cos^3{\dfrac{\pi}{9}} - 6\cos{\dfrac{\pi}{9}}$

Vì vậy: $2\cos{\dfrac{\pi}{9}}$ là nghiệm của phương trình: $x^3 - 3x - 1 = 0$

Chứng minh tương tự với $2\cos{\dfrac{5\pi}{9}}$ và $2\cos{\dfrac{7\pi}{9}}$




#317402 Tìm m để hệ sau có nghiệm thực duy nhất

Đã gửi bởi Phạm Hữu Bảo Chung on 17-05-2012 - 20:47 trong Đại số

Bạn có thể trình bày được chứ, theo mình thì như vậy là OK rồi ? :icon6:



Với một S duy nhất, một P duy nhất. Ta luôn có thể có 2 nghiệm khác nhau thỏa mãn phương trình:

$X^2 - SX + P = 0$

Do ở đây, vai trò của x và y như nhau nên trường hợp mà bạn xét không đủ dữ kiện để nói rằng, hệ có nghiệm duy nhất!

Bạn thử lại xem... Với $S = -1; P = \dfrac{-3}{2}$

Hệ có 2 cặp nghiệm phân biệt :)



#333985 Giải PT : $(8sin^{3}x+1)^{3}-162 sinx +27= 0$

Đã gửi bởi Phạm Hữu Bảo Chung on 10-07-2012 - 13:48 trong Phương trình, Hệ phương trình Lượng giác

Bài 2:
$3(1+\frac{\cos{2x}}{\cos^2{x}})^4+ 4\tan^6{x} = 7$

Giải

Phương trình ban đầu tương đương:
$3(1 + \dfrac{\cos^2{x} - \sin^2{x}}{\cos^2{x}})^4 + 4\tan^6{x} = 7$

$\Leftrightarrow 3(2 - \tan^2{x})^4 + 4\tan^6{x} = 7 \,\, (2)$

Đặt $a = \tan^2{x} \geq 0$. Phương trình (2) trở thành:
$3(2 - a)^4 + 4a^3 = 7 \Leftrightarrow 3a^4 - 20a^3 + 72a^2 - 96a + 41 = 0$

$\Leftrightarrow (a - 1)^2(3a^2 - 14a + 41) = 0$

$\Rightarrow \left[\begin{array}{l} a = 1\\3a^2 - 14a + 41 = 0 \,\, (VN)\end{array}\right.$


$\Rightarrow \tan^2{x} = 1 \Leftrightarrow \tan{x} = \pm 1$

$\Rightarrow \left[\begin{array}{l} x = \dfrac{\pm \pi}{4} + 2k\pi\\x = \dfrac{\pm 3\pi}{4} + 2k\pi\end{array}\right.$



#313252 Cho $x-y\geq 1$. Tìm giá trị lớn nhất của $P=\frac{4...

Đã gửi bởi Phạm Hữu Bảo Chung on 29-04-2012 - 08:12 trong Đại số

Cho $x-y\geq 1$. Tìm giá trị lớn nhất của $P=\frac{4}{x}-\frac{1}{y}$

Bài toán trên đủ dữ kiện chưa nhỉ.
Dễ thấy nếu chọn cặp giá trị $(x; y) = (1; - k);\;\;(k \in (0; 1])$.
$\Rightarrow$ P không tồn tại giá trị Max.
Chẳng hạn:
$(x; y) = (1; - 0,25) \Rightarrow P = 8$


$(x; y) = (1; - 0,05) \Rightarrow P = 24$

Giá trị tuyệt đối của k càng nhỏ, P càng lớn. Do đó, không tồn tại giá trị lớn nhất của biểu thức P.

Mong bạn xem lại đề.



#317394 Tìm m để hệ sau có nghiệm thực duy nhất

Đã gửi bởi Phạm Hữu Bảo Chung on 17-05-2012 - 20:39 trong Đại số

Tìm m để hệ sau có nghiệm thực duy nhất $\left\{\begin{matrix} x^2+xy+y^2=m+6 & \\ 2x+xy+2y=m& \end{matrix}\right.$

Giải

Đây là hệ đối xứng loại 1.
Do đó, nếu phương trình có một nghiệm $(x_0; y_0)$ thì nó cũng có nghiệm $(y_0; x_0)$.
Vì vậy, hệ nói trên có nghiệm thực duy nhất khi $x = y$

Khi đó, hệ ban đầu tương đương:
$\left\{\begin{array}{l}3x^2 = m + 6\\x^2 + 4x = m\end{array}\right.$


Điều kiện để hệ nói trên có nghiệm:
$\left\{\begin{array}{l}m + 6 \geq 0\\2^2 + m \geq 0\\\pm\sqrt{\dfrac{m + 6}{3}} = -2 \pm \sqrt{m + 4}\end{array}\right.$


$\Leftrightarrow \left\{\begin{array}{l}m \geq -4\\\left[\begin{array}{l} \sqrt{\dfrac{m + 6}{3}} = -2 + \sqrt{m + 4}\\- \sqrt{\dfrac{m + 6}{3}} = -2 + \sqrt{m + 4}\\- \sqrt{\dfrac{m + 6}{3}} = -2 - \sqrt{m + 4}\end{array}\right.\end{array}\right.$

$\Leftrightarrow \left\{\begin{array}{l}m \geq -4\\\left[\begin{array}{l} \sqrt{\dfrac{m + 6}{3}} + 2= \sqrt{m + 4}\\2 = \sqrt{\dfrac{m + 6}{3}} + \sqrt{m + 4}\\\sqrt{\dfrac{m + 6}{3}} = 2 + \sqrt{m + 4}\end{array}\right.\end{array}\right.$

$\Leftrightarrow \left\{\begin{array}{l}m \geq -4\\\left[\begin{array}{l} 2\sqrt{\dfrac{m + 6}{3}} = \dfrac{m - 3}{3} \,\,\, (1)\\-\dfrac{2m + 3}{3} = \sqrt{\dfrac{(m + 6)(m + 4)}{3}} \,\,\, (2)\\-\dfrac{m + 9}{3} = 2\sqrt{m + 4} \,\,\, (3)\end{array}\right.\end{array}\right.$

Ta có:
$(1) \Leftrightarrow \left\{\begin{array}{l}m \geq 3\\m^2 - 18m - 63 = 0 \end{array}\right. \Leftrightarrow m = 21$

$(2) \Leftrightarrow \left\{\begin{array}{l}m \leq \dfrac{-3}{2}\\m^2 - 18m - 63 = 0\end{array}\right. \Leftrightarrow m = -3$

$(3)$ vô nghiệm do $\forall \, m\geq - 4 \Rightarrow \dfrac{-m - 9}{3} < 0 \leq 2\sqrt{m + 4}$

Vậy $\left[\begin{array}{l} m = - 3\\m = 21\end{array}\right.$

* Với m = -3, hệ ban đầu tương:
$\left\{\begin{array}{l}x^2 + xy + y^2 = 3\\2x + xy + 2y = -3\end{array}\right. \Rightarrow \left\{\begin{array}{l}(x + y)^2 + 2(x + y) = 0\\x^2 + xy + y^2 = 3\end{array}\right.$

$\Rightarrow \left[\begin{array}{l}x = - y\\x = -y - 2\end{array}\right.$
Thế các giá trị này vào hệ ban đầu, ta thấy không thỏa mãn có nghiệm thực duy nhất.

* Với m = 21, hệ ban đầu tương đương:
$\left\{\begin{array}{l}x^2 + xy + y^2 = 27\\2x + 2y + xy = 21\end{array}\right. \Leftrightarrow \left\{\begin{array}{l}(x + y)^2 + 2(x + y) - 48 = 0\\x^2 + xy + y^2 = 27\end{array}\right.$


$\Leftrightarrow \left\{\begin{array}{l}x^2 + xy + y^2 = 27 \\\left[\begin{array}{l}x = -y - 8\\x = 6 - y\end{array}\right.\end{array}\right.$
Thế các giá trị này vào, giải ra, ta thấy hệ có nghiệm duy nhất x = y = 3.
Vậy với m = 21, hệ có nghiệm duy nhất!



#446036 Cho $a,b,c\in [1;2]$ .Tìm min của $P=\frac{(a+b...

Đã gửi bởi Phạm Hữu Bảo Chung on 29-08-2013 - 00:12 trong Bất đẳng thức và cực trị

Bài 3
Giải
Từ giả thiết, suy ra: $\left (\dfrac{a}{c} \right )^3 + \left (\dfrac{b}{c} \right )^3 = 1$
Đặt $x = \dfrac{a}{c}; y = \dfrac{b}{c}, t = x + y$. Ta có: $x^3 + y^3 = 1$.
Khi đó: $\left\{\begin{matrix}xy = \dfrac{(x + y)^3 - 1}{3(x + y)} > 0\\t^3 \leq 4(x^3 + y^3) = 4\end{matrix}\right. \Leftrightarrow \left\{\begin{matrix}xy = \dfrac{t^3 - 1}{3t} > 0\\1 < t \leq \sqrt[3]{4}\end{matrix}\right. $
 
Ta có:
$P = \dfrac{\left (\dfrac{a}{c} \right )^2 + \left (\dfrac{b}{c} \right )^2 - 1}{\left ( 1 - \dfrac{a}{c}\right )\left ( 1 - \dfrac{b}{c}\right )} = \dfrac{x^2 + y^2 - 1}{(1 - x)(1 - y)}$
 
$= \dfrac{(x + y)^2 - 2xy - 1}{1 - (x + y) + xy} = \dfrac{t^2 - 2\dfrac{t^3 - 1}{3t} - 1}{1 - t + \dfrac{t^3 - 1}{3t}}$ 
 
$= \dfrac{t^3 - 3t + 2}{(t - 1)^3} = \dfrac{t + 2}{t - 1}$
 
Xét hàm số $f(t) = \dfrac{t + 2}{t - 1}$ trên $(1; \sqrt[3]{4}]$ có $f'(t) = \dfrac{-3}{(t - 1)^2} < 0$
Vậy, hàm nghịch biến trên $(1; \sqrt[3]{4}]$
Do đó: $P = f(t) \geq f(\sqrt[3]{4}) = \dfrac{\sqrt[3]{4} + 2}{\sqrt[3]{4} - 1}$
Dấu "=" xảy ra khi $t = \sqrt[3]{4}$ và $x = y$. Khi đó: $c = \sqrt[3]{2}a = \sqrt[3]{2}b$



#446035 Cho $a,b,c\in [1;2]$ .Tìm min của $P=\frac{(a+b...

Đã gửi bởi Phạm Hữu Bảo Chung on 28-08-2013 - 23:54 trong Bất đẳng thức và cực trị

Bài 1
Giải
Đặt $x = \dfrac{c}{a + b}$, do $a, b, c \in [1; 2]$ nên suy ra $x \in \left [ \dfrac{1}{4}; 1\right ]$
Khi đó, ta có:
$P =\dfrac{(a + b)^2}{c^2 + 4ab + 4c(a + b)} =\dfrac{1}{\left (\dfrac{c}{a + b} \right )^2 + \dfrac{4ab}{(a + b)^2} + \dfrac{4c}{a + b}}$
 
$\geq \dfrac{1}{\left (\dfrac{c}{a + b} \right )^2 + 1 + \dfrac{4c}{a + b}} = \dfrac{1}{x^2 + 4x + 1}$
 
Xét hàm số $f(x) = \dfrac{1}{x^2 + 4x + 1}$ trên $\left [ \dfrac{1}{4}; 1\right ]$ có $f'(x) = \dfrac{-2(x + 2)}{(x^2 + 4x + 1)^2} < 0$ 
Hàm số đã cho nghịch biến trên $\left [ \dfrac{1}{4}; 1\right ]$
Vì vậy: $f(x) \geq f(1) = \dfrac{1}{6}$. Do đó: $P \geq \dfrac{1}{6}$. 
Dấu "=" xảy ra khi $x = 1$ và $a = b$. Khi đó: $c = 2$ và $a = b = 1$



#308884 Đề thi HSG lớp 10 trường THPT Thanh Thủy - Phú Thọ

Đã gửi bởi Phạm Hữu Bảo Chung on 07-04-2012 - 23:15 trong Thi HSG cấp Tỉnh, Thành phố. Olympic 30-4. Đề thi và kiểm tra đội tuyển các cấp.

THPT Thanh Thủy - Phú Thọ
Bài 3: (2đ) Giải hệ phương trình :
$\begin{cases} 8x^2+18y^2+36xy-(10x+15y)\sqrt{6xy} =0 \,\,\,\, (1) \\ 2x^2+3y^2=30 \,\,\,\, (2) \end{cases}$

Giải

ĐK: $xy \geq 0$
Phương trình thứ nhất của hệ tương đương:
$2(4x^2 + 9y^2 + 12xy) + 12xy - 5(2x + 3y)\sqrt{6xy} = 0$


$\Leftrightarrow 2(2x + 3y)^2 - 5(2x + 3y)\sqrt{6xy} + 12xy = 0 \,\,\,\,\,\,\,\, (1)$

Đặt $\left\{\begin{array}{l}2x + 3y = A\\\sqrt{6xy} = B \geq 0\end{array}\right.$
Phương trình (1) trở thành: $2A^2 - 5AB + 2B^2 = 0 \Leftrightarrow (2A - B)(B - 2A) = 0$

$\Leftrightarrow \left[\begin{array}{l} A = 2B\\B = 2A\end{array}\right. \Rightarrow \left[\begin{array}{l} 2x + 3y = 2\sqrt{6xy} \,\,\,\, (3)\\\sqrt{6xy} = 2(2x + 3y) \,\,\,\, (4)\end{array}\right.$

Dễ thấy ở cả hai phương trình, nếu x và y < 0 thì phương trình vô nghiệm.
Mặt khác: $xy \geq 0$. Do đó: $x, y \geq 0$.
Ta có: $(3) \Leftrightarrow (\sqrt{2x} - \sqrt{3y})^2 = 0 \Leftrightarrow 2x = 3y$.
Thế vào phương trình thứ 2 của hệ ban đầu. Giải phương trình bậc hai vừa tìm được và lấy nghiệm không âm của phương tình này.


Phương trình (4) vô nghiệm.



#293527 ĐỀ THI CHỌN HSG LỚP 10 TRƯỜNG THPT KỲ LÂM NĂM 2011 - 2012

Đã gửi bởi Phạm Hữu Bảo Chung on 12-01-2012 - 19:11 trong Thi HSG cấp Tỉnh, Thành phố. Olympic 30-4. Đề thi và kiểm tra đội tuyển các cấp.

ĐỀ THI CHỌN HSG TRƯỜNG THPT KỲ LÂM NĂM 2011 - 2012

Môn thi: Toán 10 - Thời gian: 150 phút


Câu 1. Giải các phương trình:
a, $\sqrt{x} + \sqrt[4]{17 - x^2} = 3$
b, $1 + x - 2x^2 = \sqrt{4x^2 - 1} - \sqrt{2x + 1}$

Câu 2. Giải hệ phương trình:
$\left\{\begin{array}{l}xy + y^2 + x = 7y\\\dfrac{x^2}{y} + x = 12\end{array}\right.$


Câu 3. Xét tất cả các tam thức bậc hai $f(x) = ax^2 + bx + c $ sao cho a < b và $f(x) \geq 0 \forall x$. Tìm GTNN:
$$M = \dfrac{a + b + c}{b - a}$$

Câu 4. Trên mặt phẳng tọa độ cho A(-2; -1), B(2; -4)
a, Tìm tọa độ điểm C trên Ox sao cho vecto OA và vecto CB cùng phương.
b, Tìm trên đường thẳng x = 1 điểm M sao cho $\widehat{MBA} = 45^{o}$

Câu 5. Cho các số thực dương a, b, c sao cho a + b + c = 1.
Chứng minh rằng:
$\dfrac{1}{a^2 + b^2 + c^2} + \dfrac{1}{ab} + \dfrac{1}{bc} + \dfrac{1}{ca} \geq 30$



#449477 $x^{3}+\frac{x^{3}}{(x-1)^{...

Đã gửi bởi Phạm Hữu Bảo Chung on 11-09-2013 - 21:27 trong Phương trình, hệ phương trình và bất phương trình

Bài 2

Giải

ĐK: $x \neq 1$

Phương trình đã cho tương đương:
$$x^3 + \dfrac{x^3}{(x - 1)^3} + \dfrac{3x^2}{x - 1} - 2 = 0$$

Đặt $t = x + \dfrac{x}{x - 1} = \dfrac{x^2}{x - 1}$

$\Rightarrow t^3 = x^3 + \dfrac{x^3}{(x - 1)^3} + 3\dfrac{x^2}{x - 1 }.t \Rightarrow x^3 + \dfrac{x^3}{(x - 1)^3} = t^3 – 3t^2$

 

Khi đó, phương trình trở thành: $t^3 - 3t^2 + 3t - 2 = 0 \Leftrightarrow (t - 1)^3 = 1 \Leftrightarrow t = 2$

Phương trình ban đầu vô nghiệm.




#449475 $x^{3}+\frac{x^{3}}{(x-1)^{...

Đã gửi bởi Phạm Hữu Bảo Chung on 11-09-2013 - 21:19 trong Phương trình, hệ phương trình và bất phương trình

Bài 1 có thể biến đổi như sau:

Giải

Phương trình ban đầu tương đương:

$x^2 + \dfrac{(9x)^2}{(x + 9)^2} = 40$

$\Leftrightarrow \left ( x - \dfrac{9x}{x + 9}\right )^2 + 18\dfrac{x^2}{x + 9} = 40$

$\Leftrightarrow \dfrac{x^4}{(x + 9)^2} + 18\dfrac{x^2}{x + 9} - 40 = 0$

 

Đặt $t = \dfrac{x^2}{x + 9}$, ta được: $a^2 + 18a - 40 = 0$
 

 

 




#309352 Đề thi học sinh giỏi tỉnh môn toán lớp 10 tỉnh Hà Tĩnh năm học 2011-2012

Đã gửi bởi Phạm Hữu Bảo Chung on 09-04-2012 - 22:13 trong Thi HSG cấp Tỉnh, Thành phố. Olympic 30-4. Đề thi và kiểm tra đội tuyển các cấp.

Câu 1
a) Giải phương trình: $x^2-7x+10=2\sqrt{x-2}$
b) Giải hệ phương trình:
$$\left\{\begin{matrix}x^2-y^2-2x+2y=-3
& \\y^2-2xy+2x=-4
&
\end{matrix}\right.$$


Giải

a, ĐK: $x \geq 2$
Phương trình ban đầu tương đương:
$(x - 2)(x - 5) - 2\sqrt{x - 2} = 0 \Leftrightarrow \sqrt{x - 2}[(x - 5)\sqrt{x - 2} - 2] = 0$


$\Leftrightarrow \left[\begin{array}{l} x = 2\\(x - 5)\sqrt{x - 2 } = 2 \,\,\,\,\,\,\, (2)\end{array}\right.$

Ta có: $(2) \Leftrightarrow \left\{\begin{array}{l}x \geq 5\\(x - 5)^2(x - 2) = 4\end{array}\right. \Leftrightarrow \left\{\begin{array}{l}x \geq 5\\x^3 - 12x^2 + 45x - 54 = 0\end{array}\right.$

$\Leftrightarrow \left\{\begin{array}{l}x \geq 5\\(x - 6)(x^2 - 6x + 9) = 0\end{array}\right. \Leftrightarrow \left\{\begin{array}{l}x \geq 5\\ \left[\begin{array}{l} x = 3\\x = 6\end{array}\right.\end{array}\right.$

$\Leftrightarrow x = 6$
Vậy phương trình ban đầu có 2 nghiệm: x = 6 và x = 2
b, Hơi dài.
$\left\{\begin{array}{l}x^2-y^2-2x+2y=-3\\y^2-2xy+2x=-4\end{array}\right.$

$\Leftrightarrow \left\{\begin{array}{l}(x - 1)^2 - (y - 1)^2 = -3\\(y - x)^2 - (x - 1)^2 = -5\end{array}\right.$

Đặt $\left\{\begin{array}{l}a = x - 1\\b = y - 1\end{array}\right. \Rightarrow b - a = y - x$

Phương trình ban đầu trở thành:
$\left\{\begin{array}{l}a^2 - b^2 = -3\\(b - a)^2 - a^2 = -5\end{array}\right. \Leftrightarrow \left\{\begin{array}{l}a^2 - b^2 = - 3\\b^2 - 2ab = -5\end{array}\right. \,\,\,\,(II)$

$\Leftrightarrow \left\{\begin{array}{l}5a^2 - 5b^2 = -15\,\,\,\, (1)\\3b^2 - 6ab = -15\,\,\,\, (2)\end{array}\right.$

Lấy (1) - (2) vế theo vế, ta được:
$5a^2 + 6ab - 8b^2 = 0 \Leftrightarrow (a + 2b)(5a - 4b) = 0$

$\Leftrightarrow \left[\begin{array}{l} a = -2b\\a = \dfrac{4b}{5}\end{array}\right.$
- Với a = -2b, hệ (II) trở thành:
$\left\{\begin{array}{l} a = -2b\\(-2b)^2 - b^2 = -3\end{array}\right. \Leftrightarrow \left\{\begin{array}{l} a = -2b\\b^2 = -1\end{array}\right.$

Hệ phương trình này vô nghiệm.
- Với $a = \dfrac{4b}{5}$, hệ trở thành:
$\left\{\begin{array}{l} a = \dfrac{4b}{5}\\\dfrac{16b^2}{25} - b^2 = - 3\end{array}\right. \Leftrightarrow \left\{\begin{array}{l}a = \dfrac{4b}{5}\\b^2 = \dfrac{25}{3}\end{array}\right.$


$\Leftrightarrow \left\{\begin{array}{l}a = \dfrac{4}{5}b = \pm \dfrac{4}{\sqrt{3}}\\b = \pm \dfrac{5}{\sqrt{3}}\end{array}\right.$

$\Rightarrow \left[\begin{array}{l} \left\{\begin{array}{l}x - 1 = \dfrac{4}{\sqrt{3}}\\y - 1 = \dfrac{5}{\sqrt{3}}\end{array}\right. \\ \left\{\begin{array}{l}x - 1 = \dfrac{- 4}{\sqrt{3}}\\y - 1 = \dfrac{- 5}{\sqrt{3}}\end{array}\right.\end{array}\right. \Leftrightarrow \left[\begin{array}{l} \left\{\begin{array}{l}x = 1 + \dfrac{4}{\sqrt{3}}\\y = 1 + \dfrac{5}{\sqrt{3}}\end{array}\right.\\\left\{\begin{array}{l}x = 1 - \dfrac{4}{\sqrt{3}}\\y = 1 - \dfrac{5}{\sqrt{3}}\end{array}\right.\end{array}\right.$

P/S: Đề thi 150 phút. không được sử dụng máy tính cầm tay trong phòng thi. :(



#423069 $[Giải hệ phương trình]$

Đã gửi bởi Phạm Hữu Bảo Chung on 02-06-2013 - 10:05 trong Đại số

Anh ơi, D là cái gì ?

 

$D = a_1b_2 - a_2b_1$
$Dx = c_1b_2 - c_2b_1$
$Dy = a_1c_2 - a_2c_1$




#328835 $sin2x-12(sinx-cosx)+12=0$

Đã gửi bởi Phạm Hữu Bảo Chung on 24-06-2012 - 21:45 trong Phương trình, Hệ phương trình Lượng giác

Giải phương trình

$$sin2x-12(sinx-cosx)+12=0$$

Giải

Phương trình tương đương:
$12(\sin{x} - \cos{x}) - 12 - \sin{2x} = 0$

$\Leftrightarrow (\sin^2{x} + \cos^2{x} - \sin{2x}) + 12(\sin{x} - \cos{x}) - 13 = 0$


$\Leftrightarrow (\sin{x} - \cos{x})^2 + 12(\sin{x} - \cos{x}) - 13 = 0$


$\Leftrightarrow \left[\begin{array}{l} \sin{x} - \cos{x} = 1\\\sin{x} - \cos{x} = - 13\end{array}\right.$

Vì $\sin{x}; \cos{x} \in [-1; 1] \Rightarrow -2 \leq \sin{x} - \cos{x} \leq 2$.

Do đó, ta chọn điều kiện $\sin{x} - \cos{x} = 1$.

Ta có hệ:
$\left\{\begin{array}{l}\sin{x} = \cos{x} + 1\\\sin^2{x} + \cos^2{x} = 1\end{array}\right. \Leftrightarrow \left\{\begin{array}{l}\sin{x} = \cos{x} + 1\\(\cos{x} + 1)^2 + \cos^2{x} = 1\end{array}\right.$

$\Leftrightarrow \left\{\begin{array}{l}\sin{x} = \cos{x} + 1\\2\cos{x}(\cos{x} + 1) = 0\end{array}\right. \Leftrightarrow \left\{\begin{array}{l}\sin{x} = \cos{x} + 1\\\left[\begin{array}{l} \cos{x} = 0\\\cos{x} = - 1\end{array}\right.\end{array}\right.$


$\Leftrightarrow \left[\begin{array}{l} \left\{\begin{array}{l}\cos{x} = 0\\\sin{x} = 1\end{array}\right.\\\left\{\begin{array}{l}\cos{x} = - 1\\\sin{x} = 0\end{array}\right.\end{array}\right.$

$\Rightarrow \left[\begin{array}{l}x = \dfrac{\pi}{2} + 2k\pi\\x = \pi + 2k\pi\end{array}\right. \,\, (k \in Z)$



#317992 CHỨNG MINH BẤT ĐẲNG THỨC BẰNG PHƯƠNG PHÁP HÌNH HỌC

Đã gửi bởi Phạm Hữu Bảo Chung on 19-05-2012 - 21:55 trong Các dạng toán khác

Bài 1. Cho a, b, c là các số thực dương. CMR
$\sqrt{a^{2}+b^{2}-ab}+\sqrt{b^{2}+c^{2}-bc}\geq \sqrt{a^{2}+c^{2}+ac}$

Giải

Trong hệ tọa độ Oxy, chọn:
$A = (\dfrac{a}{2}; \dfrac{\sqrt{3}a}{2}); B = (b \,; 0); C = (\dfrac{c}{2}; \dfrac{-\sqrt{3}c}{2})$


Theo Bất đẳng thức 3 điểm, ta có:
$AB + BC \geq AC$


$\Rightarrow \sqrt{(b - \dfrac{a}{2})^2 + (\dfrac{\sqrt{3}a}{2})^2} + \sqrt{(b - \dfrac{c}{2})^2 + (\dfrac{\sqrt{3}c}{2})^2} \geq \sqrt{(\dfrac{c}{2} - \dfrac{a}{2})^2 + (\dfrac{\sqrt{3}a}{2} + \dfrac{\sqrt{3}c}{2})^2}$

$\Leftrightarrow \sqrt{a^2 + b^2 - ab} + \sqrt{b^2 + c^2 - bc} \geq \sqrt{a^2 + c^2 + ac}$

Dấu "=" xảy ra khi:
Vectơ BA cùng phương với vectơ BC.
$\Rightarrow \dfrac{\dfrac{a}{2} - b}{\dfrac{\sqrt{3}a}{2}} = \dfrac{\dfrac{c}{2} - b}{\dfrac{\sqrt{3}c}{2}}$


$\Leftrightarrow \dfrac{a - 2b}{a} = \dfrac{c - 2b}{c} \Leftrightarrow 2b(a - c) = 0$
$\Leftrightarrow \left[\begin{array}{l} a = c\\b = 0\end{array}\right.$


Do a, b, c là các số thực dương nên $b \neq 0$

Thế điều kiện a = c vào đẳng thức:
$$\sqrt{a^{2}+b^{2}-ab}+\sqrt{b^{2}+c^{2}-bc} = \sqrt{a^{2}+c^{2}+ac}$$

Suy ra:
$2\sqrt{a^2 + b^2 - ab} = \sqrt{3a^2}$ $\Rightarrow a^2 - 4ab + 4b^2 = 0 \Rightarrow b = \dfrac{a}{2} = \dfrac{c}{2}$



#329293 ĐỀ TUYỂN SINH (ko chuyên) thpt CHUYÊN HÀ TĨNH 2012-2013

Đã gửi bởi Phạm Hữu Bảo Chung on 26-06-2012 - 10:51 trong Tài liệu - Đề thi

3b)Giải phương trình $\sqrt{x+1} + x + 3 = \sqrt{1-x}+3\sqrt{1-x^{2}}$

Giải

ĐK: $- 1\leq x \leq 1$
Đặt $\left\{\begin{array}{l}a = \sqrt{x + 1} \geq 0\\b = \sqrt{1 - x} \geq 0\end{array}\right. \Rightarrow \left\{\begin{array}{l}x = \dfrac{a^2 - b^2}{2}\\1 = \dfrac{a^2 + b^2}{2}\end{array}\right.$

$\Rightarrow x + 3 = \dfrac{a^2 - b^2}{2} + \dfrac{3(a^2 + b^2)}{2} = 2a^2 + b^2$


Phương trình ban đầu trở thành:
$a + 2a^2 + b^2 = b + 3ab \Leftrightarrow (2a^2 - 3ab + b^2) + a - b = 0$

$\Leftrightarrow (a - b)(2a - b + 1) = 0 \Leftrightarrow \left[\begin{array}{l} a = b\\b = 2a + 1\end{array}\right.$

$\Rightarrow \left[\begin{array}{l} \sqrt{x + 1} = \sqrt{1 - x}\\\sqrt{1 - x} = 2\sqrt{x + 1} + 1\end{array}\right. \Leftrightarrow \left[\begin{array}{l} x = 0\\1 - x = 4(x + 1) + 1 + 4\sqrt{x + 1}\end{array}\right.$

$\Leftrightarrow \left[\begin{array}{l} x = 0\\-5x - 4 = 4\sqrt{x + 1}\end{array}\right. \Leftrightarrow \left[\begin{array}{l} x = 0\\\left\{\begin{array}{l}-5x - 4 \geq 0\\25x^2 + 40x + 16 = 16(x + 1)\end{array}\right.\end{array}\right.$


$\Leftrightarrow \left[\begin{array}{l} x = 0\\\left\{\begin{array}{l}x \leq \dfrac{- 4}{5}\\x(25x + 24) = 0\end{array}\right.\end{array}\right.$

$\Leftrightarrow \left[\begin{array}{l} x = 0\\\left\{\begin{array}{l}x \leq \dfrac{-4}{5}\\\left[\begin{array}{l} x = \dfrac{-24}{25}\\x = 0\end{array}\right.\end{array}\right.\end{array}\right. \Leftrightarrow \left[\begin{array}{l} x = 0\\x = \dfrac{-24}{25}\end{array}\right.$



#286480 [Hỏi]

Đã gửi bởi Phạm Hữu Bảo Chung on 04-12-2011 - 07:07 trong Phương trình - hệ phương trình - bất phương trình

Cho:
$\left\{\begin{matrix} mx+y=2m& \\x+my=m+1 & \end{matrix}\right.$
Tìm số nguyên m để hệ phương trình trên có có nghiệm duy nhất với x, y là số nguyên

Giải

* m = 0. Hệ phương trình có nghiệm x = 1; y = 0.
* $m \neq 0$, hệ phương trình ban đầu tương đương:

$\left\{\begin{array}{l}mx + y = 2m\\mx + m^2y = m(m + 1)\end{array}\right. \Leftrightarrow \left\{\begin{array}{l}(m^2 - 1)y = m(m + 1) - 2m\\x + my = m + 1\end{array}\right.$

$\Leftrightarrow \left\{\begin{array}{l}(m - 1)(m + 1)y = m(m - 1)\\x = m + 1 - my\end{array}\right. \,\,\,\,\,\,\,\, (II)$

- Nếu m = 1, hệ (II) trở thành:
$\left\{\begin{array}{l}0y = 0\\x = 2 - y\end{array}\right.$
Hệ có vô số cặp nghiệm nguyên (x; y) = (2 - t; t)

- Nếu $m \neq 1$, hệ (II) tương đương:
$\left\{\begin{array}{l}y = \dfrac{m }{m + 1}\\x = m + 1 - m.\dfrac{m }{m + 1} = \dfrac{2m + 1}{m + 1} = 1 + y\end{array}\right.$


Do đó $y \in Z \Rightarrow x \in Z$.
Ta thấy: $y \in Z \Leftrightarrow \dfrac{m}{m + 1} \in Z \Leftrightarrow 1 - \dfrac{1}{m + 1} \in Z$

$\Rightarrow m + 1 \in U_{(1)} \Rightarrow \left[\begin{array}{l} m + 1 = 1\\m + 1 = -1\end{array}\right. \Leftrightarrow \left[\begin{array}{l} m = 0 (Loai)\\m = - 2 (tm)\end{array}\right.$

Nói tóm lại, hệ có nghiệm nguyên khi vào chỉ khi: m = 0; m = 1; m = -2



#415665 giải hệ phương trinh:$\left\{ \begin{array...

Đã gửi bởi Phạm Hữu Bảo Chung on 30-04-2013 - 21:18 trong Phương trình, hệ phương trình và bất phương trình

Em học lớp mấy nhỉ? Đây là hệ phương trình đẳng cấp bậc 2. Em có thể tham khảo tư liệu trên mạng (khá nhiều) để làm thêm những bài dạng này nhé.




#309604 ĐỀ THI CHỌN HSG LỚP 10 TRƯỜNG THPT KỲ LÂM NĂM 2011 - 2012

Đã gửi bởi Phạm Hữu Bảo Chung on 11-04-2012 - 06:00 trong Thi HSG cấp Tỉnh, Thành phố. Olympic 30-4. Đề thi và kiểm tra đội tuyển các cấp.

Câu 3. Xét tất cả các tam thức bậc hai $f(x) = ax^2 + bx + c $ sao cho a < b và $f(x) \geq 0 \forall x$. Tìm GTNN:
$$M = \dfrac{a + b + c}{b - a}$$

Cô giáo dạy Toán giải cho bọn em theo cách này, mọi người tham khảo nhé.

Giải

Do $f(x) = ax^2 + bx + c \geq 0 \,\forall x$ nên:

$f(-2) = 4a - 2b + c \geq 0$

$\Leftrightarrow a + b + c + 3(a - b) \geq 0 \Leftrightarrow a + b + c \geq 3(b - a) \,\,\,\, (I)$

Ta thấy: a < b, do đó b - a > 0. Chia hai vế của (I) cho (b - a), ta có:

$\dfrac{a + b + c}{b - a} \geq 3 \Rightarrow Min_{M} = 3$


Dấu "=" xảy ra khi $f(- 2) = 0 \Leftrightarrow 4a - 2b + c = 0 \, (a < b) $



#423114 $[Giải hệ phương trình]$

Đã gửi bởi Phạm Hữu Bảo Chung on 02-06-2013 - 12:35 trong Đại số

Ủa! Em tính chay như thế nào? Tính bằng phương pháp cộng thấy KQ cũng giống vậy mà?